3
$\begingroup$

Say we have a random process $X(t, u)$ parametrized by $t$ and $u$ that generates data $x$. We also have a prior on $u$, $p(u)$.

Am I correct in stating that the expression to find the maximum a posteriori (MAP) estimate of only $t$ would be

$$ t_{MAP} = \underset{t}{\operatorname{argmax}} \int p(x | t, u) p(u) du = \underset{t}{\operatorname{argmax}} p(x|t) $$

That is, you'd marginalize the nuisance parameters out, correct?

Now suppose $U\sim \mathcal{N}[\hat{u}, C_u]$ and $X \mid u, t \sim \mathcal{N}[M(t) u, C_x]$.

Is it correct to say that $p(x | t)$ equals the pdf of $M(t) U$? That is, $$X \mid t \sim \mathcal{N}[M(t)\hat{u}, M(t)C_uM(t)^\top]$$

Stated differently and more generally, does deriving the distribution of $f(U, t)$ (where $f$ is a bijective function) give the same result as the $\int p(x|t, u)p(u)du$?

$\endgroup$
9
  • $\begingroup$ Perhaps you benefit from having a peek at my Bayesian tutorial. You can read most of it online under "Read Sample." amazon.com/dp/B0BTNVFR65 $\endgroup$ Commented Jul 7 at 13:15
  • $\begingroup$ @RomkeBontekoe oops, fixed now! $\endgroup$ Commented Jul 7 at 13:15
  • $\begingroup$ What is $t$ here, another random parameter or a time index? $\endgroup$ Commented Jul 7 at 14:00
  • $\begingroup$ @JohandeAguas $t$ is a generic parameter. $\endgroup$ Commented Jul 7 at 15:42
  • $\begingroup$ Hi: the integral gives p(x | t). I'm not sure about the definition of f(U, t). It's not defined. It looks like possibly the joint dist of U and t ? $\endgroup$
    – mlofton
    Commented Jul 7 at 17:10

1 Answer 1

5
$\begingroup$

If $t$ is a random parameter, for which Bayesian inference is required, then $t$ should have a prior, $p(t)$, so it can have a posterior and a MAP estimate. In the most general case, $t_{MAP}=\arg\max_t p(t)\int p(x\mid u,t)p(u\mid t)du$.

If $u$ and $t$ are prior-independent, then $t_{MAP}=\arg\max_t p(t)\int p(x\mid u,t)p(u)du$.

If, on top, $p(t)$ is the uniform distribution, then $t_{MAP}=\arg\max_t \int p(x\mid u,t)p(u)du$.

$\endgroup$

Your Answer

By clicking “Post Your Answer”, you agree to our terms of service and acknowledge you have read our privacy policy.

Not the answer you're looking for? Browse other questions tagged or ask your own question.